Is set of all surjections from $mathbbN cap m+1$ to finite subset of $mathbbN$ finite? The Next CEO of Stack OverflowFinite set cardinality decreasingNumber of surjections from one set to anotherDisproving $A subset B wedge B cap C neq varnothing Rightarrow A cap C neq varnothing$Proving countabilityCardinality of the set of all involutions from $mathbb N$ to itselfproving that the set of all english words is countble.Chance of Drawing All of a SubsetAre there uncountably many surjections from $mathbb N$ to $mathbb N$?Largest separated subset of a finite setIf a set $A$ is finite then $Acap B$ is a finite set.

Why here is plural "We went to the movies last night."

Would this house-rule that treats advantage as a +1 to the roll instead (and disadvantage as -1) and allows them to stack be balanced?

How do I construct this japanese bowl?

Can the Reverse Gravity spell affect the Meteor Swarm spell?

Why do professional authors make "consistency" mistakes? And how to avoid them?

How do spells that require an ability check vs. the caster's spell save DC work?

Why does C# sound extremely flat when saxophone is tuned to G?

Return the Closest Prime Number

How to write papers efficiently when English isn't my first language?

Increase performance creating Mandelbrot set in python

How should I support this large drywall patch?

How can I open an app using Terminal?

Customer Requests (Sometimes) Drive Me Bonkers!

What does this shorthand mean?

The King's new dress

Inappropriate reference requests from Journal reviewers

Horror movie/show or scene where a horse creature opens its mouth really wide and devours a man in a stables

Which organization defines CJK Unified Ideographs?

Rotate a column

How to make a variable always equal to the result of some calculations?

Why didn't Theresa May consult with Parliament before negotiating a deal with the EU?

Visit to the USA with ESTA approved before trip to Iran

What does "Its cash flow is deeply negative" mean?

How to be diplomatic in refusing to write code that breaches the privacy of our users



Is set of all surjections from $mathbbN cap m+1$ to finite subset of $mathbbN$ finite?



The Next CEO of Stack OverflowFinite set cardinality decreasingNumber of surjections from one set to anotherDisproving $A subset B wedge B cap C neq varnothing Rightarrow A cap C neq varnothing$Proving countabilityCardinality of the set of all involutions from $mathbb N$ to itselfproving that the set of all english words is countble.Chance of Drawing All of a SubsetAre there uncountably many surjections from $mathbb N$ to $mathbb N$?Largest separated subset of a finite setIf a set $A$ is finite then $Acap B$ is a finite set.










0












$begingroup$


Set of all surjections from $1,…,m$ to $1,…,n$ is finite.

But Set of all surjections from $mathbbN$ to $mathbbN$ is not finite.

So I thought this intermediate:



Is $$ $f$$ : mathbbN cap (m+1)(=1,…,m) to$ finite subset of $mathbbN$ $wedge$ f is surjection $ $ finite set?



Thanks in advance.



Edit: I had made a serious mistake.I added word, "finite subset".










share|cite|improve this question











$endgroup$







  • 2




    $begingroup$
    There is no surjection from $1,2,...,m $ to $mathbb N$.
    $endgroup$
    – Kavi Rama Murthy
    Mar 18 at 11:47















0












$begingroup$


Set of all surjections from $1,…,m$ to $1,…,n$ is finite.

But Set of all surjections from $mathbbN$ to $mathbbN$ is not finite.

So I thought this intermediate:



Is $$ $f$$ : mathbbN cap (m+1)(=1,…,m) to$ finite subset of $mathbbN$ $wedge$ f is surjection $ $ finite set?



Thanks in advance.



Edit: I had made a serious mistake.I added word, "finite subset".










share|cite|improve this question











$endgroup$







  • 2




    $begingroup$
    There is no surjection from $1,2,...,m $ to $mathbb N$.
    $endgroup$
    – Kavi Rama Murthy
    Mar 18 at 11:47













0












0








0





$begingroup$


Set of all surjections from $1,…,m$ to $1,…,n$ is finite.

But Set of all surjections from $mathbbN$ to $mathbbN$ is not finite.

So I thought this intermediate:



Is $$ $f$$ : mathbbN cap (m+1)(=1,…,m) to$ finite subset of $mathbbN$ $wedge$ f is surjection $ $ finite set?



Thanks in advance.



Edit: I had made a serious mistake.I added word, "finite subset".










share|cite|improve this question











$endgroup$




Set of all surjections from $1,…,m$ to $1,…,n$ is finite.

But Set of all surjections from $mathbbN$ to $mathbbN$ is not finite.

So I thought this intermediate:



Is $$ $f$$ : mathbbN cap (m+1)(=1,…,m) to$ finite subset of $mathbbN$ $wedge$ f is surjection $ $ finite set?



Thanks in advance.



Edit: I had made a serious mistake.I added word, "finite subset".







elementary-set-theory






share|cite|improve this question















share|cite|improve this question













share|cite|improve this question




share|cite|improve this question








edited Mar 18 at 13:02







hina logic

















asked Mar 18 at 11:44









hina logichina logic

63




63







  • 2




    $begingroup$
    There is no surjection from $1,2,...,m $ to $mathbb N$.
    $endgroup$
    – Kavi Rama Murthy
    Mar 18 at 11:47












  • 2




    $begingroup$
    There is no surjection from $1,2,...,m $ to $mathbb N$.
    $endgroup$
    – Kavi Rama Murthy
    Mar 18 at 11:47







2




2




$begingroup$
There is no surjection from $1,2,...,m $ to $mathbb N$.
$endgroup$
– Kavi Rama Murthy
Mar 18 at 11:47




$begingroup$
There is no surjection from $1,2,...,m $ to $mathbb N$.
$endgroup$
– Kavi Rama Murthy
Mar 18 at 11:47










2 Answers
2






active

oldest

votes


















4












$begingroup$

Yes, it's empty in fact:) Note the domain is finite, and the range is countably infinite.






share|cite|improve this answer











$endgroup$












  • $begingroup$
    @CameronBuie right indeed! editing
    $endgroup$
    – Mariah
    Mar 18 at 11:55










  • $begingroup$
    I'm afraid this is complicated. $mathbbN$ is not range but codomain. But these are the same notation, hence I cannot fix it.
    $endgroup$
    – hina logic
    Mar 18 at 12:11











  • $begingroup$
    @hinalogic: "Surjection" means that you want the range to be equal to the codomain.
    $endgroup$
    – Henning Makholm
    Mar 18 at 12:12











  • $begingroup$
    @HenningMakholm Should I wrote "$1,…,m$ to subset of $mathbbN$"?
    $endgroup$
    – hina logic
    Mar 18 at 12:15







  • 1




    $begingroup$
    @hinalogic: What do you intend for the word "surjection" to achieve in your question? A "surjection from $Y$ to a subset of $X$" is just the same as a "map from $Y$ to $X$" -- there is no point in speaking of surjections unless you care what the range is.
    $endgroup$
    – Henning Makholm
    Mar 18 at 12:16



















0












$begingroup$

Since $mathbbN$ is infiite and $1,2,3,...,m$ is finite, surjections from the latter to the former are impossible. Otherwise, we would be claiming that $mathbbN$'s size is smaller or equal than $1,2,3,...,m$'s, which is absurd






share|cite|improve this answer









$endgroup$













    Your Answer





    StackExchange.ifUsing("editor", function ()
    return StackExchange.using("mathjaxEditing", function ()
    StackExchange.MarkdownEditor.creationCallbacks.add(function (editor, postfix)
    StackExchange.mathjaxEditing.prepareWmdForMathJax(editor, postfix, [["$", "$"], ["\\(","\\)"]]);
    );
    );
    , "mathjax-editing");

    StackExchange.ready(function()
    var channelOptions =
    tags: "".split(" "),
    id: "69"
    ;
    initTagRenderer("".split(" "), "".split(" "), channelOptions);

    StackExchange.using("externalEditor", function()
    // Have to fire editor after snippets, if snippets enabled
    if (StackExchange.settings.snippets.snippetsEnabled)
    StackExchange.using("snippets", function()
    createEditor();
    );

    else
    createEditor();

    );

    function createEditor()
    StackExchange.prepareEditor(
    heartbeatType: 'answer',
    autoActivateHeartbeat: false,
    convertImagesToLinks: true,
    noModals: true,
    showLowRepImageUploadWarning: true,
    reputationToPostImages: 10,
    bindNavPrevention: true,
    postfix: "",
    imageUploader:
    brandingHtml: "Powered by u003ca class="icon-imgur-white" href="https://imgur.com/"u003eu003c/au003e",
    contentPolicyHtml: "User contributions licensed under u003ca href="https://creativecommons.org/licenses/by-sa/3.0/"u003ecc by-sa 3.0 with attribution requiredu003c/au003e u003ca href="https://stackoverflow.com/legal/content-policy"u003e(content policy)u003c/au003e",
    allowUrls: true
    ,
    noCode: true, onDemand: true,
    discardSelector: ".discard-answer"
    ,immediatelyShowMarkdownHelp:true
    );



    );













    draft saved

    draft discarded


















    StackExchange.ready(
    function ()
    StackExchange.openid.initPostLogin('.new-post-login', 'https%3a%2f%2fmath.stackexchange.com%2fquestions%2f3152689%2fis-set-of-all-surjections-from-mathbbn-cap-m1-to-finite-subset-of-mathb%23new-answer', 'question_page');

    );

    Post as a guest















    Required, but never shown

























    2 Answers
    2






    active

    oldest

    votes








    2 Answers
    2






    active

    oldest

    votes









    active

    oldest

    votes






    active

    oldest

    votes









    4












    $begingroup$

    Yes, it's empty in fact:) Note the domain is finite, and the range is countably infinite.






    share|cite|improve this answer











    $endgroup$












    • $begingroup$
      @CameronBuie right indeed! editing
      $endgroup$
      – Mariah
      Mar 18 at 11:55










    • $begingroup$
      I'm afraid this is complicated. $mathbbN$ is not range but codomain. But these are the same notation, hence I cannot fix it.
      $endgroup$
      – hina logic
      Mar 18 at 12:11











    • $begingroup$
      @hinalogic: "Surjection" means that you want the range to be equal to the codomain.
      $endgroup$
      – Henning Makholm
      Mar 18 at 12:12











    • $begingroup$
      @HenningMakholm Should I wrote "$1,…,m$ to subset of $mathbbN$"?
      $endgroup$
      – hina logic
      Mar 18 at 12:15







    • 1




      $begingroup$
      @hinalogic: What do you intend for the word "surjection" to achieve in your question? A "surjection from $Y$ to a subset of $X$" is just the same as a "map from $Y$ to $X$" -- there is no point in speaking of surjections unless you care what the range is.
      $endgroup$
      – Henning Makholm
      Mar 18 at 12:16
















    4












    $begingroup$

    Yes, it's empty in fact:) Note the domain is finite, and the range is countably infinite.






    share|cite|improve this answer











    $endgroup$












    • $begingroup$
      @CameronBuie right indeed! editing
      $endgroup$
      – Mariah
      Mar 18 at 11:55










    • $begingroup$
      I'm afraid this is complicated. $mathbbN$ is not range but codomain. But these are the same notation, hence I cannot fix it.
      $endgroup$
      – hina logic
      Mar 18 at 12:11











    • $begingroup$
      @hinalogic: "Surjection" means that you want the range to be equal to the codomain.
      $endgroup$
      – Henning Makholm
      Mar 18 at 12:12











    • $begingroup$
      @HenningMakholm Should I wrote "$1,…,m$ to subset of $mathbbN$"?
      $endgroup$
      – hina logic
      Mar 18 at 12:15







    • 1




      $begingroup$
      @hinalogic: What do you intend for the word "surjection" to achieve in your question? A "surjection from $Y$ to a subset of $X$" is just the same as a "map from $Y$ to $X$" -- there is no point in speaking of surjections unless you care what the range is.
      $endgroup$
      – Henning Makholm
      Mar 18 at 12:16














    4












    4








    4





    $begingroup$

    Yes, it's empty in fact:) Note the domain is finite, and the range is countably infinite.






    share|cite|improve this answer











    $endgroup$



    Yes, it's empty in fact:) Note the domain is finite, and the range is countably infinite.







    share|cite|improve this answer














    share|cite|improve this answer



    share|cite|improve this answer








    edited Mar 18 at 11:55

























    answered Mar 18 at 11:47









    MariahMariah

    2,1431718




    2,1431718











    • $begingroup$
      @CameronBuie right indeed! editing
      $endgroup$
      – Mariah
      Mar 18 at 11:55










    • $begingroup$
      I'm afraid this is complicated. $mathbbN$ is not range but codomain. But these are the same notation, hence I cannot fix it.
      $endgroup$
      – hina logic
      Mar 18 at 12:11











    • $begingroup$
      @hinalogic: "Surjection" means that you want the range to be equal to the codomain.
      $endgroup$
      – Henning Makholm
      Mar 18 at 12:12











    • $begingroup$
      @HenningMakholm Should I wrote "$1,…,m$ to subset of $mathbbN$"?
      $endgroup$
      – hina logic
      Mar 18 at 12:15







    • 1




      $begingroup$
      @hinalogic: What do you intend for the word "surjection" to achieve in your question? A "surjection from $Y$ to a subset of $X$" is just the same as a "map from $Y$ to $X$" -- there is no point in speaking of surjections unless you care what the range is.
      $endgroup$
      – Henning Makholm
      Mar 18 at 12:16

















    • $begingroup$
      @CameronBuie right indeed! editing
      $endgroup$
      – Mariah
      Mar 18 at 11:55










    • $begingroup$
      I'm afraid this is complicated. $mathbbN$ is not range but codomain. But these are the same notation, hence I cannot fix it.
      $endgroup$
      – hina logic
      Mar 18 at 12:11











    • $begingroup$
      @hinalogic: "Surjection" means that you want the range to be equal to the codomain.
      $endgroup$
      – Henning Makholm
      Mar 18 at 12:12











    • $begingroup$
      @HenningMakholm Should I wrote "$1,…,m$ to subset of $mathbbN$"?
      $endgroup$
      – hina logic
      Mar 18 at 12:15







    • 1




      $begingroup$
      @hinalogic: What do you intend for the word "surjection" to achieve in your question? A "surjection from $Y$ to a subset of $X$" is just the same as a "map from $Y$ to $X$" -- there is no point in speaking of surjections unless you care what the range is.
      $endgroup$
      – Henning Makholm
      Mar 18 at 12:16
















    $begingroup$
    @CameronBuie right indeed! editing
    $endgroup$
    – Mariah
    Mar 18 at 11:55




    $begingroup$
    @CameronBuie right indeed! editing
    $endgroup$
    – Mariah
    Mar 18 at 11:55












    $begingroup$
    I'm afraid this is complicated. $mathbbN$ is not range but codomain. But these are the same notation, hence I cannot fix it.
    $endgroup$
    – hina logic
    Mar 18 at 12:11





    $begingroup$
    I'm afraid this is complicated. $mathbbN$ is not range but codomain. But these are the same notation, hence I cannot fix it.
    $endgroup$
    – hina logic
    Mar 18 at 12:11













    $begingroup$
    @hinalogic: "Surjection" means that you want the range to be equal to the codomain.
    $endgroup$
    – Henning Makholm
    Mar 18 at 12:12





    $begingroup$
    @hinalogic: "Surjection" means that you want the range to be equal to the codomain.
    $endgroup$
    – Henning Makholm
    Mar 18 at 12:12













    $begingroup$
    @HenningMakholm Should I wrote "$1,…,m$ to subset of $mathbbN$"?
    $endgroup$
    – hina logic
    Mar 18 at 12:15





    $begingroup$
    @HenningMakholm Should I wrote "$1,…,m$ to subset of $mathbbN$"?
    $endgroup$
    – hina logic
    Mar 18 at 12:15





    1




    1




    $begingroup$
    @hinalogic: What do you intend for the word "surjection" to achieve in your question? A "surjection from $Y$ to a subset of $X$" is just the same as a "map from $Y$ to $X$" -- there is no point in speaking of surjections unless you care what the range is.
    $endgroup$
    – Henning Makholm
    Mar 18 at 12:16





    $begingroup$
    @hinalogic: What do you intend for the word "surjection" to achieve in your question? A "surjection from $Y$ to a subset of $X$" is just the same as a "map from $Y$ to $X$" -- there is no point in speaking of surjections unless you care what the range is.
    $endgroup$
    – Henning Makholm
    Mar 18 at 12:16












    0












    $begingroup$

    Since $mathbbN$ is infiite and $1,2,3,...,m$ is finite, surjections from the latter to the former are impossible. Otherwise, we would be claiming that $mathbbN$'s size is smaller or equal than $1,2,3,...,m$'s, which is absurd






    share|cite|improve this answer









    $endgroup$

















      0












      $begingroup$

      Since $mathbbN$ is infiite and $1,2,3,...,m$ is finite, surjections from the latter to the former are impossible. Otherwise, we would be claiming that $mathbbN$'s size is smaller or equal than $1,2,3,...,m$'s, which is absurd






      share|cite|improve this answer









      $endgroup$















        0












        0








        0





        $begingroup$

        Since $mathbbN$ is infiite and $1,2,3,...,m$ is finite, surjections from the latter to the former are impossible. Otherwise, we would be claiming that $mathbbN$'s size is smaller or equal than $1,2,3,...,m$'s, which is absurd






        share|cite|improve this answer









        $endgroup$



        Since $mathbbN$ is infiite and $1,2,3,...,m$ is finite, surjections from the latter to the former are impossible. Otherwise, we would be claiming that $mathbbN$'s size is smaller or equal than $1,2,3,...,m$'s, which is absurd







        share|cite|improve this answer












        share|cite|improve this answer



        share|cite|improve this answer










        answered Mar 18 at 11:58









        asdfasdf

        1973




        1973



























            draft saved

            draft discarded
















































            Thanks for contributing an answer to Mathematics Stack Exchange!


            • Please be sure to answer the question. Provide details and share your research!

            But avoid


            • Asking for help, clarification, or responding to other answers.

            • Making statements based on opinion; back them up with references or personal experience.

            Use MathJax to format equations. MathJax reference.


            To learn more, see our tips on writing great answers.




            draft saved


            draft discarded














            StackExchange.ready(
            function ()
            StackExchange.openid.initPostLogin('.new-post-login', 'https%3a%2f%2fmath.stackexchange.com%2fquestions%2f3152689%2fis-set-of-all-surjections-from-mathbbn-cap-m1-to-finite-subset-of-mathb%23new-answer', 'question_page');

            );

            Post as a guest















            Required, but never shown





















































            Required, but never shown














            Required, but never shown












            Required, but never shown







            Required, but never shown

































            Required, but never shown














            Required, but never shown












            Required, but never shown







            Required, but never shown







            Popular posts from this blog

            Lowndes Grove History Architecture References Navigation menu32°48′6″N 79°57′58″W / 32.80167°N 79.96611°W / 32.80167; -79.9661132°48′6″N 79°57′58″W / 32.80167°N 79.96611°W / 32.80167; -79.9661178002500"National Register Information System"Historic houses of South Carolina"Lowndes Grove""+32° 48' 6.00", −79° 57' 58.00""Lowndes Grove, Charleston County (260 St. Margaret St., Charleston)""Lowndes Grove"The Charleston ExpositionIt Happened in South Carolina"Lowndes Grove (House), Saint Margaret Street & Sixth Avenue, Charleston, Charleston County, SC(Photographs)"Plantations of the Carolina Low Countrye

            random experiment with two different functions on unit interval Announcing the arrival of Valued Associate #679: Cesar Manara Planned maintenance scheduled April 23, 2019 at 00:00UTC (8:00pm US/Eastern)Random variable and probability space notionsRandom Walk with EdgesFinding functions where the increase over a random interval is Poisson distributedNumber of days until dayCan an observed event in fact be of zero probability?Unit random processmodels of coins and uniform distributionHow to get the number of successes given $n$ trials , probability $P$ and a random variable $X$Absorbing Markov chain in a computer. Is “almost every” turned into always convergence in computer executions?Stopped random walk is not uniformly integrable

            How should I support this large drywall patch? Planned maintenance scheduled April 23, 2019 at 00:00UTC (8:00pm US/Eastern) Announcing the arrival of Valued Associate #679: Cesar Manara Unicorn Meta Zoo #1: Why another podcast?How do I cover large gaps in drywall?How do I keep drywall around a patch from crumbling?Can I glue a second layer of drywall?How to patch long strip on drywall?Large drywall patch: how to avoid bulging seams?Drywall Mesh Patch vs. Bulge? To remove or not to remove?How to fix this drywall job?Prep drywall before backsplashWhat's the best way to fix this horrible drywall patch job?Drywall patching using 3M Patch Plus Primer